Three Semicircles

Geometry Level 4

Two semicircles of the same radius R R are inscribed inside a semicircle of radius 1 1 with no overlap. What's the largest possible R R ?


The answer is 0.6098902004274422.

This section requires Javascript.
You are seeing this because something didn't load right. We suggest you, (a) try refreshing the page, (b) enabling javascript if it is disabled on your browser and, finally, (c) loading the non-javascript version of this page . We're sorry about the hassle.

3 solutions

Michael Mendrin
Dec 28, 2019

Borrowing Vreken's labelling, in this figure the variable is the slope s s of the line C B CB . The radius O D OD of the semicircle is 1 1 , and r r is the radius of the semicircles to be maximized. \;\; Δ C B Q \Delta CBQ is a right triangle.

Letting O O be the origin with coordinates ( 0 , 0 ) (0,0) , then the coordinates of C C is

( C x , C y ) = ( 1 r r C s c ( θ ) , 0 ) (Cx, Cy) = (1-r-rCsc(\theta),\; 0)

The coordinates of P P is then

( P x , P y ) = ( 1 r r C s c ( θ ) + r C o s ( θ ) , r S i n ( θ ) ) (Px, Py) = ( 1-r-rCsc(\theta) + rCos(\theta), \; rSin(\theta))

so that

( 1 r r C s c ( θ ) + r C o s ( θ ) ) 2 + ( r S i n ( θ ) ) 2 = 1 r \sqrt { {(1-r-rCsc(\theta)+rCos(\theta))}^2 + {(rSin(\theta))}^2 } = 1-r

If s = T a n ( θ ) s = Tan(\theta) , then

r = 2 s ( 1 s + s 2 ) 2 s ( 1 s + s 2 ) + ( 1 2 s + 2 s 2 ) 1 + s 2 r=\dfrac { 2s\left( 1-s+{ s }^{ 2 } \right) }{ 2s\left( 1-s+{ s }^{ 2 } \right) +(1-2s+2{ s }^{ 2 })\sqrt { 1+{ s }^{ 2 } } }

The maximum value for r r occurs when s s is the real solution to the following cubic equation

3 s 3 3 s 2 + 2 s 1 = 0 3s^3-3s^2+2s-1=0

or s = 1 3 ( 1 ( 2 5 + 29 ) 1 3 + ( 1 2 ( 5 + 29 ) ) 1 3 ) s = \dfrac{1}{3} \left( 1- \left( \dfrac{2}{5+\sqrt{29}} \right)^{\frac{1}{3}} + \left( \dfrac{1}{2} \left(5 + \sqrt{29} \right) \right) ^{\frac{1}{3}} \right)

Plugging this value for s s into the expression for r r delivers the approximate answer 0.6098902004274422 0.6098902004274422

Nice solution! I think you have some typos, though.

( 1 r r csc θ + r cos θ ) 2 + ( sin θ ) 2 = 1 r \sqrt{(1 - r - r \csc \theta + r \cos \theta)^2 + (\sin \theta)^2} = 1 - r should be ( 1 r r csc θ + r cos θ ) 2 + ( r sin θ ) 2 = 1 r \sqrt{(1 - r - r \csc \theta + r \cos \theta)^2 + (r \sin \theta)^2} = 1 - r (there's an r r with the sin θ \sin \theta )

s = 1 3 ( 1 ( 2 5 + 29 ) 1 3 ) + ( 1 2 ( 5 + 29 ) 1 3 ) s = \frac{1}{3}(1 - (\frac{2}{5 + \sqrt{29}})^{\frac{1}{3}}) + (\frac{1}{2}(5 + \sqrt{29})^{\frac{1}{3}}) should be s = 1 3 ( 1 ( 2 5 + 29 ) 1 3 + ( 1 2 ( 5 + 29 ) ) 1 3 ) s = \frac{1}{3}(1 - (\frac{2}{5 + \sqrt{29}})^{\frac{1}{3}} + (\frac{1}{2}(5 + \sqrt{29}))^{\frac{1}{3}}) (some misplaced parentheses)

David Vreken - 1 year, 5 months ago

Thanks for the catch. Both times! I goofed when I had mis-advised Patrick Corn.

Needless to say, my original notes isn't anywhere near as concise as my posted answer. Most of the hassle was coming up with the cubic for s s .

Michael Mendrin - 1 year, 5 months ago

Reminded me of that Y!A question of cutting out the 2 curve of maximum volume cone from a 10x20 metal sheet.

Saya Suka - 7 months, 2 weeks ago
David Vreken
Dec 27, 2019

Label the diagram as follows, draw A O AO and P Q PQ , let y y be the radius of the small semi-circle (so that C P = A P = Q B = Q D = y CP = AP = QB = QD = y ), let x = C O x = CO , let z = P B z = PB , and let θ = O Q P \theta = \angle OQP . Since A O = O D = 1 AO = OD = 1 and A P = Q D = y AP = QD = y , P O = O Q = 1 y PO = OQ = 1 - y , and since P O Q \triangle POQ is an isosceles triangle, O P Q = O Q P = θ \angle OPQ = \angle OQP = \theta , and by the exterior angle theorem of O P Q \triangle OPQ , P O C = 2 θ \angle POC = 2\theta . Since B B is a point of tangency, B \angle B is a right angle.

By Pythagorean's Theorem on C Q B \triangle CQB , ( y + z ) 2 + y 2 = ( x + 1 y ) 2 (y + z)^2 + y^2 = (x + 1 - y)^2 , which rearranges to z = ( x + 1 ) ( x 2 y + 1 ) y z = \sqrt{(x + 1) (x - 2 y + 1)} - y .

By Pythagorean's Theorem on P Q B \triangle PQB , P Q = y 2 + z 2 PQ = \sqrt{y^2 + z^2} .

By the law of cosines on O P Q \triangle OPQ , cos θ = ( y 2 + z 2 ) 2 + ( 1 y ) 2 ( 1 y ) 2 2 ( 1 y ) y 2 + z 2 = y 2 + z 2 2 ( 1 y ) \cos \theta = \frac{(\sqrt{y^2 + z^2})^2 + (1-y)^2 - (1-y)^2}{2(1-y)\sqrt{y^2 + z^2}} = \frac{\sqrt{y^2 + z^2}}{2(1 - y)} .

By the law of cosines on O P C \triangle OPC , cos 2 θ = ( 1 y ) 2 + x 2 y 2 2 x ( 1 y ) \cos 2\theta = \frac{(1-y)^2 + x^2 - y^2}{2x(1-y)} .

Using cosine double angle formula cos 2 θ = 2 cos 2 θ 1 \cos 2 \theta = 2 \cos^2 \theta - 1 and combining the above equations gives ( 1 y ) ( ( 1 y ) 2 + x 2 y 2 ) = x y 2 + x ( ( x + 1 ) ( x 2 y + 1 ) y ) 2 2 x ( 1 y ) 2 (1 -y)((1 -y)^2 + x^2 - y^2) = xy^2 + x(\sqrt{(x + 1)(x - 2y + 1)} - y)^2 - 2x(1 - y)^2 , which can be found numerically to have a maximum y y -value of y 0.60989 y \approx \boxed{0.60989} .

Thanks for reconfirming my original answer. As I said to Patrick Corn, I do have the exact expression for this, but it's very long. Maybe I will post that exact expression.

I'll try to post my solution to this problem, which is considerably less elegant than yours.

Michael Mendrin - 1 year, 5 months ago

Deleting my solution now. Thanks for pointing out my error.

Patrick Corn - 1 year, 5 months ago


7 r 3 12 r 2 + 8 r 2 = 0 h a s o n e r e a l s o l u t i o n . r = 0.60687...... 7r^3-12r^2+8r-2=0~has ~one~ real~solution. ~~~~r=0.60687......

It's incorrect because your last purple equation should be 0 = r(7r³+4r²+8r-2)

Saya Suka - 7 months, 2 weeks ago

1 pending report

Vote up reports you agree with

×

Problem Loading...

Note Loading...

Set Loading...